¿Qué es la materia oscura?

Para cumplir con los requisitos establecidos requeriría (1) una publicación del tamaño de un libro y (2) una comprensión más profunda de lo que yo personalmente tengo. Sin embargo, dado que Quora no es el lugar adecuado para escribir un libro de todos modos , puedo saber lo suficiente como para tomar una decisión razonable sobre esto. Dicho esto, sin embargo, si eres un experto real en una de estas áreas y entiendes algo que he explicado incorrectamente, ¡házmelo saber en los comentarios para que pueda solucionarlo!

Sí, todavía es bastante largo, pero tenga en cuenta que esta es la versión corta .

¿Qué es la materia oscura (DM)?

La materia oscura es solo materia que no interactúa a través de las fuerzas electromagnéticas (EM) o fuertes nucleares. Sin interacciones EM significa que no puede emitir luz, ni absorber luz, ni reflejar, refractar o dispersar luz de ninguna manera. Esto, naturalmente, hace que sea bastante difícil de ver (por lo tanto, materia “oscura”, aunque supongo que es más “transparente” que “oscura”). Nuestras mejores mediciones actuales indican que algo así como el 85% de la materia en nuestro Universo observable (aproximadamente una cuarta parte del contenido total de masa-energía) es materia oscura.

¿No es la materia oscura extraña / espeluznante?

De ningún modo. Los neutrinos, por ejemplo, satisfacen la definición de Dark Matter, solo representan una fracción tan pequeña del DM total en el Universo que las personas tienden a descuidarlos cuando preguntan: “¿De qué está hecha la Dark Matter?”

No hay nada extraño o inusual en ciertas partículas que no interactúan de ciertas maneras. Los neutrones no tienen carga eléctrica (aunque tienen propiedades EM, pero eso no es ni aquí ni allá), y los electrones no interactúan a través de la Fuerza Fuerte, entonces, ¿por qué no debería haber partículas que interactúen con ninguno, como los neutrinos? Decir que interactuar con la luz es “normal” es un sesgo puramente humano, porque confiamos mucho en la vista. Tener muchos DM en el Universo no es de ninguna manera “extraño”; ¡solo significa que el Universo no gira en torno a lo que la humanidad considera conveniente!

¿Por qué estamos seguros de que existe DM?

De ninguna manera es una lista completa, pero debería dar una idea de los tipos de evidencia que tenemos. Cada uno de estos tomaría al menos un capítulo de un libro para explicarlo adecuadamente, pero con suerte esto dará una idea general.

  • Curvas de rotación galáctica .

    Cuando un objeto orbita a otro, el objeto en órbita tiene que estar constantemente acelerando hacia el objeto central (o, más precisamente, ambos aceleran hacia su centro de masa combinado). Sin esa aceleración, el cuerpo en órbita simplemente volaría.

    Cuanto más rápido se mueve el cuerpo en órbita, más aceleración se necesita para mantenerlo en su órbita. Como en este caso la aceleración se debe a la gravedad, esto significa que la masa central tiene que ser más grande. Para una órbita circular de un objeto pequeño [matemática] m [/ matemática] a una distancia [matemática] r [/ matemática] y velocidad [matemática] v [/ matemática] alrededor de un objeto grande (y supuesto estacionario) [matemática] M [ / matemáticas], el requisito de aceleración da

    [matemáticas] \ frac {v ^ 2} {r} = \ frac {GM} {r ^ 2} [/ matemáticas]

    que a su vez nos da la relación

    [matemáticas] v = \ sqrt {\ frac {GM} {r}} [/ matemáticas].

    (Estoy haciendo esto con la gravedad newtoniana por simplicidad; hacerlo con rigor completo requeriría relatividad general. En esta situación, la aproximación newtoniana es en general bastante buena).

    Para un objeto más complicado que solo dos partículas puntuales, siempre que haya suficiente simetría, la versión gravitacional de la ley de Gauss dice que la [matemática] M [/ matemática] relevante es la masa total de todo en la galaxia que está a una distancia menor que [matemáticas] r [/ matemáticas] desde el centro .

    [ Editado para tener en cuenta: Para que esto sea exactamente correcto, la distribución de la materia tendría que ser esféricamente simétrica, que no son las galaxias. Como resultado, los cálculos reales son un poco más complicados de lo que se muestra aquí.]

    Esto nos permite “pesar” diferentes partes de la galaxia, midiendo la relación entre [matemáticas] r [/ matemáticas] y [matemáticas] v [/ matemáticas]. (Podemos medir las velocidades de rotación comparando los desplazamientos al rojo en los lados de la galaxia que se aproximan y retroceden).

    Esta imagen de Wiki muestra el resultado de esta medición:

La línea “esperado del disco visible” se determina sumando las masas de todas las partes de la galaxia que podemos ver. (Cómo medimos esa masa es una discusión completamente diferente).

  • Lente gravitacional.

    En la relatividad general, cada vez que la luz pasa a través de un campo gravitacional, ese campo dobla ligeramente su camino. Esto actúa como una lente gravitacional y puede producir, por ejemplo, “Anillos de Einstein”, como esta imagen de Wiki:

El “anillo” es una imagen distorsionada de una sola galaxia azul ubicada detrás de la galaxia roja en el centro. La luz de la galaxia azul se apaga en todas las direcciones, pero la gravedad de la galaxia roja la dobla. Esto significa que la luz que comienza en un “camino directo” hacia nosotros nunca nos alcanza, pero la luz que originalmente nos faltaba en una cantidad específica (en cualquier dirección) se dobla hacia nosotros, lo que hace que parezca que proviene un montón de direcciones diferentes, lo que resulta en la imagen del anillo que se ve aquí.

Este es un ejemplo muy dramático de lentes gravitacionales, pero hay efectos mucho más sutiles que aún pueden ser útiles. En la lente gravitacional débil, el análisis estadístico de las distorsiones en la luz que recibimos nos permite “trazar” el campo gravitacional entre nosotros y las galaxias distantes. A menudo, esto solo muestra más masa de la que sabemos explicar, pero eso podría explicarse simplemente asumiendo que nuestra comprensión de la gravedad está desactivada.

Sin embargo, hay algo más que es mucho más difícil de explicar de esa manera: el Bullet Cluster.

(Imagen de A Matter of Fact en nasa.gov)

¿Que está pasando aqui? Bueno, dos cúmulos de galaxias chocaron entre sí, y esto es lo que siguió. La coloración roja representa dónde está la materia visible , mientras que la coloración azul representa dónde está la materia oscura , como se infiere por la lente gravitacional. ¿Por qué están tan separados? Bueno, la mayor parte de la materia luminosa en un cúmulo de galaxias está en el medio Intracluster, un plasma caliente y denso. Cuando estos plasmas chocan entre sí, una cantidad significativa de la materia se ralentiza. Sin embargo, dado que Dark Matter interactúa solo muy débilmente, los componentes de DM de los dos grupos fueron libres de pasar sin impedimentos, lo que resultó en una separación (como se ve aquí). No solo hay materia luminosa “no suficiente” … ¡está en el lugar equivocado! Un pequeño número de científicos sigue comprometido a encontrar formas de explicar esto sin DM, y han tenido un éxito parcial , pero solo al incluir cosas aún no medidas que son mucho más extrañas que DM (por ejemplo, un tensor de rango 3 campo, que, si bien es posible, sería el primer campo tensorial de un rango tan alto jamás encontrado).

  • Efecto sobre el fondo cósmico de microondas.

    Durante los primeros cientos de miles de años después del Big Bang, El Universo estuvo lo suficientemente caliente como para estar altamente ionizado, lo que lo hizo más o menos opaco a la luz; los fotones se movían como cualquier otra partícula. Sin embargo, una vez que las cosas se enfriaron lo suficiente, cantidades significativas de protones y electrones se combinaron en Hidrógeno neutro, que es (más o menos) transparente para la mayor parte de la luz que existía en ese momento. Esto sucedió bastante rápido (en términos de tiempo cosmológico), por lo que fue como si toda la luz pinball alrededor de todo el Universo se liberara repentinamente de una vez, capturando efectivamente una instantánea del Universo en ese momento de su evolución.

    Dado que esta luz se lanzó en todas partes del Universo , podemos apuntar nuestros radiotelescopios en cualquier dirección que queramos, y ahí está: el fondo cósmico de microondas (CMB). Es casi la misma temperatura en todas las direcciones, pero hay pequeñas diferencias (generalmente alrededor de una parte en [matemáticas] 10 ^ 5 [/ matemáticas]), y hemos medido estas pequeñas variaciones con una precisión extraordinaria: primero a través del satélite COBE, que luego fue reemplazado por el WMAP más avanzado, que luego fue reemplazado por el Planck (nave espacial) más avanzado, que está actualmente en funcionamiento.

    Estas pequeñas variaciones pueden decirnos mucho sobre el universo primitivo. Por ejemplo, los análisis estadísticos de estas variaciones muestran la firma distintiva de las ondas de presión que se propagan a través de ese plasma temprano, y la naturaleza de estas oscilaciones acústicas de Baryon puede decirnos mucho sobre qué tipo de cosas existían. Específicamente, los protones y electrones serían arrastrados por los fotones (dominantes), convirtiéndose en parte de la onda, pero Dark Matter no lo haría , y solo se verían afectados indirectamente por los pequeños cambios resultantes en la gravedad. La presencia y abundancia de Dark Matter, por lo tanto, afecta cómo estas ondas impactan las variaciones de temperatura en el CMB.

  • La formación de estructura a gran escala.

    La historia estándar dada en las explicaciones científicas populares es así: el Universo comenzó caliente y denso y más o menos uniforme, luego se expandió, enfrió y se agrupó en estrellas y galaxias. Sin embargo, esta historia es incompleta, lo que significa que las galaxias no existirían sin materia oscura .

    A nivel superficial, la historia tiene sentido; diablos, llegué casi a la mitad de un doctorado. en Física sin notar ningún problema con ella! Suena tan plausible debido a cómo funciona la gravedad: si la materia se distribuye de manera más o menos uniforme, pero algunos lugares son un poco más densos que otros, la gravedad tenderá a agrandar y aumentar esas sobredensidades. ¿Por qué? Bueno, incluso si una región es un poco más densa que sus vecinas, seguirá ganando el tira y afloja gravitacional y gradualmente acumulará más y más masa. Por supuesto, una vez que tiene más masa, gana el tira y afloja aún más , por lo que es un proceso de fuga que termina en grandes grupos unidos gravitacionalmente.

    ¿Entonces, cuál es el problema? Bueno, considera el aire en la habitación contigo ahora mismo. ¿Hay pequeñas variaciones de densidad? Por supuesto, ya que la uniformidad perfecta es imposible. Pero, ¿se está formando en grupos exponencialmente más densos? ¡Ciertamente no! La razón de esto es que, bajo este tipo de condiciones, cuando la densidad de un gas aumenta, también lo hace su presión . ¡Esa presión hace que la región demasiado densa se expanda hacia afuera nuevamente, devolviendo la densidad al promedio!

    Ahora, por supuesto, las escalas y temperaturas involucradas son totalmente diferentes. Una gran región de gas tendrá más atracción gravitacional que una pequeña bolsa de aire más denso en su habitación, y el gas en el espacio tampoco necesita estar a temperatura ambiente. Entonces, si el gas puede enfriarse lo suficiente, eso puede reducir la presión lo suficiente como para que gane la gravedad . Pero, cuanto más se comprime, más se calienta, porque está convirtiendo la energía gravitacional en energía térmica, por lo que la presión aumenta nuevamente. Esto significa que formar una galaxia es un proceso muy gradual, durante el cual tiene que deshacerse constantemente de enormes cantidades de energía. Si fuera solo una nube de gas, sin ninguna interferencia externa, este proceso no sería lo suficientemente rápido, y hoy no tendríamos galaxias.

    Pero, como saben, las cosas calientes emiten calor mucho más rápido que las frías. Entonces, si queremos que las galaxias se formen para el día de hoy (o, de hecho, antes de que la expansión del espacio haga que la materia sea demasiado diluida para formar galaxias), algo tiene que estar obligando al gas a comprimirse y volverse más denso y más caliente de lo que sería poder bajo su propia gravedad. Entrar: materia oscura. Debido a que solo interactúa débilmente, Dark Matter no tiene presión como el gas . Entonces, el argumento sobre el proceso gravitacional de fuga realmente funciona para Dark Matter. DM no puede deshacerse de la energía con mucha facilidad, por lo que la conservación de la energía y el momento angular significa que solo puede colapsar a unas 200 veces la densidad de fondo, pero el halo de materia oscura resultante proporciona un pozo de gravedad suficiente para “sembrar” el formación de galaxias visibles. Entonces, no es una coincidencia que las curvas rotacionales galácticas mostraran grandes cantidades de materia oscura … ¡las galaxias no se habrían formado allí sin ella!

    Como resultado de este proceso de “siembra” de DM, los modelos teóricos y las simulaciones por computadora de la formación de estructuras de DM han tenido bastante éxito al describir las propiedades estadísticas de cómo se distribuyen las galaxias ahora, así como cómo se distribuyeron anteriormente en la historia de El Universo (que podemos medir mirando galaxias muy distantes).

Bien, por eso creemos que existe Dark Matter. Pero, la siguiente pregunta obvia es … bueno … ¿qué es ? ¿De qué está hecho ? ¿Cuáles son sus propiedades? Aquí, tenemos un conocimiento muy parcial y múltiples teorías diferentes, cualquier combinación de las cuales podría ser correcta (o, tal vez, ninguna de ellas). Esto nos lleva a:

¿Qué sabemos sobre las propiedades de Dark Matter?

Una vez más, esto no es exhaustivo, pero debería darle una idea decente.

  1. Hace frío”.

    Esta es la razón por la cual el modelo dominante actual de cosmología se llama el modelo Lambda-CDM: “Lambda” ([matemática] \ Lambda [/ matemática]) representa la constante cosmológica, y “CDM” significa “Materia oscura fría”.

    Cuando un astrofísico describe algo como “frío”, generalmente significa que la velocidad térmica asociada es mucho menor que la velocidad de la luz. Según este estándar, el aire en Death Valley es “frío”. Pero, nuevamente, para los cosmólogos, las galaxias son básicamente partículas puntuales, ¡así que todo es cuestión de escala y perspectiva!

    Entonces, ¿por qué DM necesita estar “frío”? Bueno, recuerde que la agrupación de DM fue parte integral de la formación de estructuras como las galaxias. Sin embargo, si la DM estuviera muy caliente (y, por lo tanto, las partículas se movían muy rápido), esto evitaría que se aglomere correctamente. Lo he explicado en términos muy vagos, pero los efectos se comprenden matemáticamente bien. De hecho, es similar a algo que sucede con los fotones: amortiguación de difusión (o “amortiguación de seda”). Sin embargo, en el caso de Dark Matter, el resultado sería una demora significativa (o incluso la prevención total) de la formación de galaxias, en la medida en que Dark Matter puede descartarse por observación.

    Por cierto, así es como sabemos que no se trata solo de neutrinos: dado lo que sabemos sobre el Universo temprano, ¡estarían demasiado calientes!

  2. Solo interactúa muy débilmente.

    Esto es parte de la definición de Dark Matter, pero es bueno ver la confirmación de observación. Conozco menos detalles sobre esto, pero sí sé que existen límites de observación en la “sección transversal de interacción” de Dark Matter, tanto en términos de sus interacciones con la materia luminosa como por sus procesos teorizados de autoaniquilación (en los que dos partículas de DM podrían interactuar y aniquilarse entre sí). Además, como se discutió anteriormente, el Bullet Cluster muestra halos de materia oscura gigantes más o menos simplemente pasándose uno al otro, lo que sugiere interacciones muy débiles.

Entonces … dadas esas propiedades,

¿De qué podría estar hecho Dark Matter?

Hay dos teorías principales (que yo sepa) que sugieren la existencia de tipos específicos de partículas nuevas. Ambos están bien motivados teóricamente (como en, tenemos buenas razones para sospechar que podrían existir partículas con esas propiedades particulares), pero ninguno ha sido confirmado experimentalmente todavía. Curiosamente, las dos partículas predichas son totalmente diferentes entre sí , no ligeras variaciones sobre el mismo tema.

Al final del día, cualquiera de estas teorías podría ser correcta, o ambas (dada la existencia de neutrinos, no hay necesidad de que todo el resto de la DM sea un solo tipo de partícula), o, por supuesto, tampoco.

Entonces, ¿cuáles son las teorías?

  1. Axions

    Me siento moralmente obligado a poner este primero, a pesar de que el otro es más popular en este momento, porque mi universidad está muy involucrada en el Experimento Axion Dark Matter (ADMX), ¡y por supuesto estoy apoyando a mis colegas!

    La existencia de Axions se ha teorizado desde la década de 1970, pero solo recientemente tenemos la tecnología para comenzar a tratar de medirlos en el laboratorio. Los axiones son partículas extremadamente pequeñas (a diferencia de los WIMP, la otra opción líder), por lo que tendrían que existir en cantidades realmente enormes . Aún así, debido a que interactúan tan débilmente, es difícil detectarlos, incluso con miles de millones de ellos pasando a través de su detector en una pequeña fracción de segundo.

    Los artículos wiki vinculados hacen un mejor trabajo de explicar la motivación teórica y la búsqueda experimental de lo que podría. Es realmente bastante elegante y resolvería muchos misterios sobresalientes en la física de partículas (como el Problema Strong CP), pero no puedo hacerle justicia.

  2. Partículas masivas de interacción débil (WIMP)

    (Referencia de Wiki: Partículas masivas que interactúan débilmente)

    Para explicar por qué los WIMP son teóricamente atractivos, tengo que desviarme un poco hacia la “abundancia de reliquias”, es decir, cuántas partículas de un tipo dado quedaron después de que el Universo se enfriara y generalmente se convirtiera en un lugar más estable.

    En el universo primitivo, todo era muy denso y muchos tipos diferentes de partículas estaban “estrechamente acopladas” (es decir, interactuaban entre sí con frecuencia). Sin embargo, a medida que el universo se hizo más grande y más frío, estas tasas de interacción disminuyeron, más o menos hasta detenerse, un fenómeno conocido como “congelamiento”. El momento en que algo “se congela” depende de una serie de cosas, incluida su masa y la intensidad con la que interactúa con otras cosas.

    Este “congelamiento” tiene un efecto enorme en la abundancia de varias partículas en el Universo. Por ejemplo, considere el proceso

    [matemáticas] n \ Leftrightarrow p + e ^ – + \ overline {\ nu_e} [/ math],

    en el que un neutrón puede convertirse en un protón, un electrón y un antineutrino de electrones (o viceversa). Si retrocede lo suficiente, esta reacción estará en equilibrio termodinámico, al igual que con cualquier reacción química que pueda ir en cualquier dirección. Sin embargo, una vez que los neutrinos se “congelan”, el equilibrio ya no se puede mantener (aunque todavía pueden ocurrir otros procesos, como la desintegración beta). La abundancia relativa de protones y neutrones en el momento del congelamiento está determinada por dos factores: la diferencia de masa [matemática] \ Delta m [/ matemática] entre las dos partículas y la temperatura del Universo cuando ocurrió el congelamiento. Estos factores se combinan para determinar por cuánto se favorece termodinámicamente la partícula más ligera, con una dependencia exponencial:

    [matemáticas] \ frac {N_n} {N_p} \ propto e ^ {- \ Delta mc ^ 2 / k T} [/ matemáticas]

    donde [math] k [/ math] es la constante de Boltzmann.

    Esto claramente afecta la “abundancia de reliquias” de las partículas involucradas (aquí, protones y neutrones). Por lo tanto, en general, la abundancia de una partícula dada en el Universo actual está significativamente influenciada por la masa de esa partícula y la intensidad con la que interactúa (ya que eso afecta el tiempo de congelación y, por lo tanto, la temperatura de congelación).

    Esto nos lleva al llamado “milagro WIMP”: una partícula que interactúa predominantemente a través de la Fuerza Nuclear Débil, y que tiene una masa cercana a la escala de masa asociada con interacciones Débiles ([matemáticas] \ sim 100 \ text {GeV} / c ^ 2 [/ math]), tendría una abundancia de reliquias que coincidiría con la abundancia medida de Dark Matter en el Universo. Dado que tal partícula ya se especulaba que existía (en el contexto de la supersimetría), esto era muy teóricamente atractivo para muchas personas, aunque nuestra incapacidad para encontrarla hasta ahora ha amortiguado algunos de sus espíritus.

Entonces, ahí lo tienes: mi resumen de Dark Matter. ¡Espero que valiera la pena el tiempo que tomó leer! (Y mucho menos escribir …)

¿Qué es la materia oscura? Nadie lo sabe a ciencia cierta.

Sin embargo, tenemos algunas pistas sobre su composición. Aquí hay un resumen de lo que sabemos:

La primera evidencia de la existencia de materia oscura se produjo cuando la astrónoma Vera Rubin refinó una técnica para determinar las velocidades de rotación de las estrellas dentro de las galaxias espirales. Después de que ella y su equipo representaron todo el material visible dentro de las galaxias (polvo, gas, estrellas), resultó que las estrellas en las porciones externas de las galaxias giraban a velocidades mucho más altas de lo esperado.

En esta imagen de una galaxia, la línea superior son las velocidades medidas de las estrellas dentro de esa galaxia. Observe cómo se mueven cada vez más rápido a medida que uno se aleja del núcleo interno brillante y presumiblemente masivo de la galaxia. La línea inferior representa las velocidades que se esperaría que fueran visibles, siendo la única cosa que actúa gravitacionalmente. Es de esperar que, a medida que las hipotéticas estrellas se alejen del centro masivo de la galaxia, se muevan más lentamente. Pero ellos no.

Al parecer, había algo más allí. Ese algo tenía masa pero no apareció en ninguna encuesta de todo el espectro electromagnético. Este “lo que sea” recibió el nombre de ” materia oscura “.

Entonces, esto nos dice lo primero que sabemos al respecto: existe.

¿Qué es eso?

¿Son partículas invisibles, similares a los neutrinos, que no interactúan con el resto del universo de forma electromagnética (produciendo así la luz)? ¿O es la gravedad de alguna manera extraña a distancias muy grandes? (Una hipótesis conocida como Dinámica Newtoniana Modificada: MOND)

En un intento por responder a esta pregunta, los astrónomos han ideado detectores y han tomado imágenes de cúmulos de galaxias para observar cómo funciona la materia oscura (partículas o MOND).

La gravedad, según la relatividad general, es la deformación del espacio. Como tal, un objeto masivo tenderá a doblar la luz a su alrededor como una lente. Los astrónomos pueden obtener imágenes de cúmulos de galaxias muy masivas y observar el efecto de deformación que tienen sobre la luz proveniente de galaxias distantes más allá. Este efecto se conoce como lente gravitacional.

En esta imagen del telescopio espacial Hubble de un cúmulo de galaxias masivo, las delgadas bandas de luz en forma de arco son las imágenes deformadas de las galaxias en la lejana distancia, más allá del cúmulo de galaxias.

Al medir la cantidad de deformación, los astrónomos pueden determinar la masa del grupo. Al obtener imágenes del cúmulo, los astrónomos pueden determinar cuánta materia “normal” hay.

Resulta que esto, y casi todos los demás, el cúmulo de galaxias es mucho más masivo de lo que se puede explicar con solo mirar cuántas estrellas y cuánto gas y polvo hay en este cúmulo. De hecho, la materia visible es una porción muy pequeña de la masa total observada a través de lentes gravitacionales.

Esto nos lleva a la segunda cosa que sabemos sobre la materia oscura: es más común que la “materia normal”.

A medida que se realizaban estudios de tales grupos masivos, los astrónomos comenzaron a poder resolver, utilizando lentes gravitacionales, la distribución de la materia oscura. Resulta que los astrónomos no pueden calcular la cantidad de flexión que hay, pero también pueden calcular, más o menos, dónde está la materia oscura en el cúmulo de la misma manera que se puede determinar la forma de una gota de vidrio mirando la forma en que dobla la luz.

Un estudio destacado comparó la distribución aparente de la materia oscura, desde la lente, hasta la distribución de hidrógeno caliente entre las galaxias. El hidrógeno caliente impregna el espacio intergaláctico y tiene una cierta cantidad de masa por sí solo. La pregunta era: ¿seguiría la materia oscura más o menos las mismas “reglas” que la materia normal? Aquí hay un resultado:

Esta es una imagen de un par de cúmulos de galaxias que salen de una colisión larga y lenta.

En esta imagen se superponen dos manchas de colores. La gota roja es la distribución medida del hidrógeno intergaláctico. Las manchas azules muestran evidencia de dónde está la materia oscura, a través de lentes gravitacionales. Esto se resuelve restando el efecto de la materia normal visible.

Lo sorprendente es que la materia oscura no parece tener la misma distribución que la materia normal. Además, se comporta de manera diferente. Mientras que el hidrógeno se “deja atrás” un poco después de la colisión, en otras palabras, el hidrógeno colisiona consigo mismo y se ralentiza mientras que la materia estelar más densa y menos difusa se mueve alegremente, la distribución de la materia oscura parece seguir la distribución promedio del racimo.

Esto nos dice lo tercero que sabemos sobre la materia oscura: no interactúa con la materia regular o con sí misma (mucho) excepto gravitacionalmente. Además, actúa “en caliente”, es decir, “movimiento rápido”, ya que no se acumula mucho en escalas pequeñas, pero no es “relativísticamente caliente”, ya que se mueve a un porcentaje significativo de la velocidad de la luz.

Las encuestas de galaxias nos han permitido determinar que la cantidad de materia oscura no siempre parece tener mucho que ver con la cantidad de materia normal en una galaxia.

En muchos casos, por ejemplo, las galaxias enanas pequeñas y de baja luminosidad tienen mucha más materia oscura en relación con su cantidad aparente de materia normal que las galaxias mucho más grandes.

Esto nos dice la cuarta cosa que sabemos: la materia oscura probablemente no sea el resultado de algún truco extraño de que la gravedad está jugando a grandes distancias. (Pero, eso sigue siendo una posibilidad remota).

Finalmente, los científicos han intentado, usando varias técnicas, detectar la interacción de la materia oscura con la materia normal de la misma manera que los neutrinos, partículas que solo interactúan de mala gana con la materia normal (se dice que un neutrino puede pasar a través de una luz año de plomo sin colisión) fueron detectados.

Aquí está el interior de un dispositivo que usa una de esas técnicas. Es un tanque masivo de agua ultra pura. Los astrónomos buscan evidencia de materia oscura buscando destellos de luz emitidos cuando las partículas chocan con el fluido. Desafortunadamente, los astrónomos no saben exactamente lo que están buscando. Esto se complica por el hecho de que los neutrinos también, en raras ocasiones, colisionan con partículas en el fluido, lo que dificulta el análisis de qué eventos son neutrinos y qué eventos son “materia oscura”.

Los globos alrededor de los bordes son tubos fotomultiplicadores. (Tenga en cuenta los dos técnicos en la imagen para la escala).

Estas técnicas, que son demasiado variadas y detalladas para delinear en una sola publicación de Quora, han tenido un único resultado, para usar un término técnico: bupkis *

  • * bupkis: sustantivo. Nada, Zilch, Nothing, Niente, Crickets …

Esto nos lleva a lo último que sabemos sobre la materia oscura, es realmente distante.

¿Así que qué es lo?

Aún no lo sabemos.

Tracemos el viaje que llevó al concepto de materia oscura a convertirse en un principio fundamental de la física. La pregunta de qué es será respondida cuando comencemos a rastrear los fundamentos de los conceptos detrás de ella.

La historia comienza en 1932, cuando el astrónomo holandés Jan Oort comenzó a rastrear el movimiento de las estrellas en nuestro vecindario galáctico local. Analizando las velocidades con las que parecían moverse, Oort concluyó que la masa de nuestra galaxia parecía ser bastante más de lo que se pensaba. Sin embargo, sus medidas fueron encontradas erróneas.

Al año siguiente, en 1933, el astrónomo suizo Fritz Zwicky estudió el movimiento de las galaxias en un cúmulo llamado Coma Cluster. Una vez más, la masa de la galaxia calculada simplemente mirando las galaxias y estimando sus masas no parecía funcionar, ya que el movimiento de las galaxias en este cúmulo parecía implicar que el cúmulo era mucho más masivo que la masa calculada real.

(El cúmulo de galaxias Coma)

Otro resultado interesante que se produjo en la misma década fortaleció aún más el reclamo de que haya una masa invisible en la mayoría de las galaxias. En 1939, Horace Babcock informó en su Ph.D. La tesis de que la curva de rotación (es decir, un gráfico de la velocidad de rotación de la galaxia sobre sí misma trazada contra la distancia desde el centro galáctico) de la galaxia de Andrómeda no parecía obedecer la ley de órbitas de Kepler. La figura muestra la expectativa de la ley de Kepler en rojo y el valor real en verde.

Una tercera evidencia (una relativamente más reciente) vino en forma de lentes gravitacionales. Este es el fenómeno en el que los rayos de luz se doblan por la presencia de un objeto masivo (más precisamente, un campo gravitacional fuerte). Por lo tanto, este objeto masivo actúa efectivamente como una lente , que puede enfocar la luz hacia usted y hacer que tome un camino completamente diferente.
Mediante un análisis de lentes, se descubre que los grupos galácticos son mucho más masivos de lo que sugiere su componente visible.
(Los arcos circulares alrededor de la burbuja central NO son galaxias, son después de imágenes causadas por lentes gravitacionales).

¿Todavía no estás convencido de que algo anda mal?

Quizás la ley de gravitación de Newton y las leyes de Kepler no se mantienen a grandes distancias, después de todo. Entonces las curvas de rotación son, meh, no lo suficientemente buenas.

Tal vez la lente significó algo misterioso, pero no puedo aceptar sin ambigüedad una teoría radicalmente nueva solo para encajar en una observación.

La prueba final y definitiva fue un análisis del Bullet Cluster. Este gran tipo involucra a dos grupos galácticos que chocan. Eso está obligado a producir una fuerte radiación. El análisis de rayos X mostró que la mayor parte de la radiación provenía del centro de la colisión. Sin embargo, el análisis de lentes mostró que la mayor parte de la masa yacía fuera de lo que era visible.

(Una imagen compuesta del Bullet Cluster. Observe la emisión brillante de la región similar a “Bullet”).

El único remedio para esto parecía ser el postulado de que había algo que tenía masa, pero que no era visible para los telescopios que causaba esta anomalía.

Este “Algo” se denominó “Materia oscura”.

Ahora descubramos qué significa la “Oscuridad”. Oscuro significaría coloquialmente la ausencia de luz. En física, la fuerza responsable de la emisión de luz es la interacción electromagnética.

Por lo tanto, una posible definición de materia oscura sería:

“Un tipo hipotético de materia que no es visible con telescopios pero que representa la mayor parte de la materia en el universo”.

Yo agregaría “y no interactúa a través de la fuerza electromagnética”. Constituye no menos del 26.8% de nuestro universo observable.

Ahora viene la pregunta, “¿Qué constituye la materia oscura?”
Varias teorías han intentado responder a esta pregunta, pero a partir de hoy no existe una respuesta definitiva al problema de la materia oscura.

Para que no haya dudas de que la física ha tendido a adoptar una teoría sofisticada para explicar el problema, me gustaría destacar que existen varias teorías como la Dinámica Newtoniana Modificada (MOND) y la Gravedad Tensor-Vector-Escalar (TeVeS) para rectificar el problema. problemas destacados anteriormente. Sin embargo, ninguno de ellos explica adecuadamente el problema con las masas de clúster Galaxy.

La física de partículas ofrece soluciones listas para los astrofísicos que buscan el componente ideal de partículas candidatas de la materia oscura. Los neutrinos son candidatos cada vez más populares para la materia oscura (de un tipo muy específico llamado materia oscura caliente). La búsqueda de otro tipo de materia llamada “materia oscura fría” continúa. Esto lleva a teorías sobre partículas masivas de interacción débil (WIMP), objetos de halo compactos masivos (MACHO), modelos teóricos de cuerdas que involucran axiones, etc. para explicar su estructura.

Para lectura adicional:

1. https://en.wikipedia.org/wiki/Co
2. https://en.wikipedia.org/wiki/Da
3. http://w.astro.berkeley.edu/~mwh
4. https://icecube.wisc.edu/outreac
5. https://en.wikipedia.org/wiki/We
6. https://en.wikipedia.org/wiki/Axion
7. https://en.wikipedia.org/wiki/Ma

Disculpas por la larga respuesta.

La mayor parte del universo está compuesto de energía oscura, una fuerza misteriosa que impulsa la expansión acelerada del universo. El siguiente ingrediente más grande es la materia oscura, que solo interactúa con el resto del universo a través de su gravedad. La materia normal, incluidas todas las estrellas, planetas y galaxias visibles, constituye menos del 5 por ciento de la masa total del universo.
Los astrónomos no pueden ver la materia oscura directamente, pero pueden estudiar sus efectos. Pueden ver la luz doblada por la gravedad de los objetos invisibles (llamados lentes gravitacionales). También pueden medir que las estrellas orbitan alrededor de sus galaxias más rápido de lo que deberían.

Todo esto puede explicarse si hubiera una gran cantidad de materia invisible atada en cada galaxia, lo que contribuye a su masa general y velocidad de rotación.

Los astrónomos saben más sobre lo que no es la materia oscura que sobre lo que es.

La materia oscura es oscura: no emite luz y no se puede ver directamente, por lo que no puede ser estrellas o planetas.

La materia oscura no es nubes de materia normal: las partículas de materia normal se llaman bariones. Si la materia oscura estuviera compuesta de bariones, sería detectable a través de la luz reflejada. [Galería: materia oscura en todo el universo]

La materia oscura no es antimateria: la antimateria aniquila la materia al contacto, produciendo rayos gamma. Los astrónomos no los detectan.

La materia oscura no es agujeros negros: los agujeros negros son lentes de gravedad que doblan la luz. Los astrónomos no ven suficientes eventos de lentes para dar cuenta de la cantidad de materia oscura que debe existir.

Estructura en el universo formada en las escalas más pequeñas primero. Se cree que la materia oscura se condensó primero para formar un “andamiaje”, con materia normal en forma de galaxias y cúmulos que siguen las concentraciones de materia oscura.

Los científicos están utilizando una variedad de técnicas en todas las disciplinas de astronomía y física para buscar materia oscura:

  • Colisionadores de partículas como el Gran Colisionador de Hadrones.
  • Instrumentos de cosmología como WMAP y Planck.
  • Experimentos de detección directa que incluyen CDMS, XENON, Zeplin, WARP, ArDM y otros.
  • Experimentos de detección indirecta que incluyen: detectores de rayos gamma (Fermi desde el espacio y telescopios Cherenkov desde el suelo); telescopios de neutrinos (IceCube, Antares); detectores de antimateria (Pamela, AMS-02) e instalaciones de radiografías y radiografías.

    (Fuente: Dark Matter Mystery Explained: A Reader’s Guide (Infographic)

Veo que esta pregunta recibió muchas respuestas agradables, pero ninguna parece mencionar un concepto muy importante: la ecuación de estado.

A los cosmólogos les gusta mantener las cosas simples. En lo que respecta a la evolución de la geometría del universo, está determinada por una relación muy simple: la ecuación de estado, que relaciona la presión y la densidad de energía de las “cosas” que llenan el universo.

La densidad de energía es, bueno, la energía es la fuerza por la distancia, la densidad significa dividir por volumen, por lo que es la fuerza por la distancia sobre el volumen, que es lo mismo que la fuerza sobre el área. Lo cual es lo mismo que presión. Entonces la densidad de energía y la presión se miden usando las mismas unidades. Lo que significa que su razón es un número puro que no tiene unidades. Este número a menudo se denota con minúsculas [math] w [/ math] en la literatura.

Así que aquí está la cosa: el valor de [matemáticas] w [/ matemáticas] solo les dice a los volúmenes sobre la expansión y la evolución geométrica del universo.

Cuando [math] w = 0 [/ math], no hay presión; Los cosmólogos lo llaman “polvo”.

Para la mayoría de los asuntos con los que estamos familiarizados, con temperaturas y presiones muy por debajo de los valores relativistas, [matemática] 0 [/ matemática]. Sin embargo, no es todo polvo: la presión significa compresión, lo que significa ondas de sonido, lo que significa la disipación de la energía cinética. Cuando las cosas son lo suficientemente calientes y densas como para ser relativistas, [matemáticas] w = 1/3 [/ matemáticas]. Esto sería un gas relativista o “gas de fotones”. Se cree que el universo primitivo estaba dominado por el gas fotón, pero el gas fotónico se diluye muy rápidamente, por lo que hoy su contribución al universo es insignificante.

Y luego está la famosa constante cosmológica: si lo vemos como una cosa, su ecuación de estado es bastante exótica, [matemática] w = -1 [/ matemática]. ¡Esa es una gran presión negativa!

Entonces, la pregunta se convierte en esta: ¿Cuál es la proporción de [matemáticas] w = 0 [/ matemáticas], [matemáticas] 0 [/ matemáticas] y [matemáticas] w = -1 [/ matemáticas] cosas que componen el universo ?

Cada uno de estos componentes responde de manera diferente a un universo en expansión que tiene algunas ondas iniciales o fluctuaciones en la densidad. En cosas con [matemática] w = 0 [/ matemática], estas fluctuaciones crecen más allá del límite: las regiones que eran densas para empezar se vuelven cada vez más densas, las regiones que eran menos densas se diluyen aún más.

En cosas con [matemáticas] 0 [/ matemáticas], como mencioné, las ondas de sonido / disipación juegan un papel, por lo que la presencia de este tipo de cosas ralentiza el crecimiento de fluctuaciones / perturbaciones. En cuanto a las cosas con [matemáticas] w = -1 [/ matemáticas], su respuesta a la gravedad es peculiar. Para la mayoría de las cosas, siendo la gravedad atractiva, ralentiza la expansión. ¡No es así cuando [matemáticas] w = -1 [/ matemáticas]: la presión es negativa, significa que la gravedad actúa como si fuera repulsiva! (No parezca demasiado sorprendido: las burbujas también se elevan en el agua, en respuesta a la gravedad.) No solo eso, sino que las cosas con [matemáticas] w = -1 [/ matemáticas] nunca se diluyen: a medida que el universo se expande, la energía La densidad de estas cosas permanece constante. Lo que significa que eventualmente dominará sobre otras cosas (que se diluyen) y cuando eso suceda, la expansión se acelerará.

Entonces, el ejercicio se vuelve bastante fácil. Dados todos los datos cosmológicos que tenemos disponibles, ¿cuál será la proporción de cosas con [matemáticas] w = 0 [/ matemáticas], cosas con [matemáticas] 0 [/ matemáticas] y cosas con [matemáticas] w = -1 [ /matemáticas]? Y la respuesta es (aproximadamente) 25%, 5% y 70%, respectivamente.

Sabemos cuál es el 5%: es todo lo normal que conocemos. El 25% es polvo pero no puede estar hecho de materia normal. Por lo tanto, no sabemos qué es, pero al darle un nombre, suena más bien informado de lo que realmente somos, por lo que lo llamamos “materia oscura”. Lo mismo ocurre con [matemáticas] w = -1 [/ matemáticas]: realmente no sabemos qué es, pero al llamarlo “energía oscura”, ¡suena muy inteligente!

OK, realmente no puedo hacer justicia real a las complejas teorías detrás de la cosmología física, pero espero que esto explique por qué pensamos que todas estas “cosas oscuras” deben existir y lo poco que realmente sabemos sobre ellas.

Wheeler señaló que el espacio es solo marginalmente menos denso en energía que la materia, aproximadamente 185 GeV / cm ^ 3. El se basa en interacciones de vacío con la materia. La opinión popular es que el espacio exhibe espontáneamente fluctuaciones aleatorias. Sin embargo, el espacio es una noción relativa, no absoluta, la distancia entre dos cosas puede ser cualquier cosa dependiendo de su velocidad relativa. Nadie ha visto energía que se origina espontáneamente desde algún punto en el espacio.

Podemos considerar que el espacio se compone de intervalos relativos de espacio-tiempo debido a las interacciones de la materia ordinaria. El espacio es tiempo en el sentido que es el retraso antes de que un evento pueda afectar a otro sin violar el orden de causas y efectos. Cada evento exhibe la hora local t = h / e y la distancia x = c / (h * f) para cada participante relativamente.

Podemos preguntar si es razonable esperar que haya suficiente energía en el universo para exhibir la energía de punto cero (ZPE) del vacío. Si comenzamos con un modelo simple de un universo en expansión. Si consideramos la densidad de energía del espectro de energía que vemos hoy de la energía de hoy, podemos aproximarnos a la magnitud general de la energía que se ha desplazado al rojo y aleatorizado a lo que vemos como la energía de punto cero del vacío hoy.

A las energías de vacío, vemos el doble de la densidad de energía de los fotones que tienen la mitad de la energía. A medida que la energía de cada fotón se aproxima a cero, la densidad de energía de los fotones de esa energía se aproxima al infinito. Hay aproximadamente cuatro veces más fotones de la mitad de la energía. Si no fuera por la naturaleza cuántica de la energía, de hecho, esperaríamos que la energía del vacío podría ser infinita, como algunos suponen.

Dado que podemos explicar razonablemente la energía del vacío, no hay razón para postular la fluctuación del vacío como una propiedad inherente del vacío que sale de la nada. En lugar de que el espacio nazca mágicamente, puede considerarse que está compuesto de energía que viene de todas las direcciones desde los comienzos del universo.

De acuerdo con la relatividad general, la luz cerca de grandes masas, como los centros galácticos y los cúmulos de galaxias, toma un camino más largo que atraviesa el pozo de gravedad de las masas cercanas y parece ser relativamente más lenta al pasar una regla estándar, considerando que la velocidad de la luz se mide localmente por un reloj más lento. Esto es relativamente más lento que nuestra luz exhibe masa en reposo, ya que consiste en energía en un marco de reposo de luz más lento de la misma manera que un fotón atrapado en una caja espejada se agrega a la masa en reposo de la caja por e = m * c ^ 2. Y esta ralentización del tiempo ES el efecto gravitacional, deformando el espacio en una dimensión de tiempo.

A medida que el universo se expande y la densidad de energía relativa disminuye, los habitantes de esa época parecen tener casi la misma consistencia a lo largo del tiempo. Dado que la materia ralentiza el tiempo dada una cantidad casi constante de materia, el tiempo se acelera relativamente a medida que el universo se expande.

Así como la velocidad de la luz siempre es constante localmente, también lo es la distancia. No importa cuán rápido o lento sea su tiempo en relación con otra persona, la luz siempre tomará un nanosegundo para pasar su regla. Si su reloj es lento relativamente, se deduce que su luz también es relativamente lenta.

A medida que el universo se expande, el espacio de tiempo se expande y la densidad de masa disminuye relativamente, y por lo tanto el tiempo y la velocidad de la luz son relativamente más rápidos, pero no se notaría localmente porque la luz aún pasa una regla en un nanosegundo.

Con esta comprensión, la materia oscura y la energía pueden entenderse como la falta de uniformidad en esta expansión, más lento cerca de los centros galácticos donde se concentra la masa llamamos a ese efecto materia oscura, y más rápido en el espacio profundo donde llamamos al efecto energía oscura.

En la mecánica cuántica no hay tiempo, todo es atemporal. La velocidad del tiempo es en efecto infinita. Excepto por el tiempo de desaceleración de la materia, podríamos esperar que no hubiera tiempo. La materia manifiesta el tiempo al exhibir un orden local inmutable de eventos (t = h / e) que distingue a nuestro mundo de todos los mundos posibles.

Ver también

La respuesta de Jim Whitescarver a Basado en las teorías de la relatividad, incluida la gravedad y la dilatación del tiempo, ¿cómo sería un gráfico de la “velocidad del tiempo” sobre la vida del universo en relación con nuestra velocidad del tiempo actual?

La respuesta de Jim Whitescarver a ¿Cuál es la naturaleza del vacío?

La respuesta de Jim Whitescarver a ¿Pueden los efectos de la materia oscura y la energía oscura ser explicados solo por la relatividad general?

La materia, como sabemos, está compuesta de protones, neutrones y electrones. Los protones y los neutrones se unen en núcleos y los átomos son núcleos rodeados por un complemento completo de electrones. El hidrógeno está compuesto de un protón y un electrón. El helio está compuesto por dos protones, dos neutrones y dos electrones. El carbono está compuesto de seis protones, seis neutrones y seis electrones. Los elementos más pesados, como el hierro, el plomo y el uranio, contienen cantidades aún mayores de protones, neutrones y electrones. Todo el material compuesto por protones, neutrones y electrones se conoce como “materia bariónica”.

Hasta hace unos treinta años, los astrónomos pensaban que el universo estaba compuesto casi por completo de esta “materia bariónica”, átomos ordinarios. Sin embargo, en las últimas décadas, se ha acumulado cada vez más evidencia que sugiere que hay algo en el universo que no podemos ver, tal vez alguna nueva forma de materia. La masa que los astrónomos infieren para las galaxias, incluida la nuestra, es aproximadamente diez veces mayor que la masa que se puede asociar con las estrellas, el gas y el polvo en una galaxia. Esta discrepancia de masa ha sido confirmada por observaciones de lentes gravitacionales, la curvatura de la luz predicha por la teoría de la relatividad general de Einstein.

Al medir los movimientos de las estrellas y el gas, los astrónomos pueden estimar la masa de las galaxias. En nuestro propio sistema solar, podemos usar la velocidad de la Tierra alrededor del Sol para medir la masa del Sol. (La Tierra se mueve alrededor del Sol a 30 kilómetros por segundo. Si el Sol fuera cuatro veces más masivo, entonces la Tierra necesitaría moverse alrededor del Sol a 60 kilómetros por segundo para que permanezca en su órbita.) El Sol se mueve alrededor de la Vía Láctea a 225 kilómetros por segundo. Podemos usar esta velocidad para medir la masa de nuestra galaxia. Del mismo modo, las observaciones ópticas y de radio de gas y estrellas en galaxias distantes permiten a los astrónomos determinar la distribución de masa en estos sistemas.

Además, las galaxias en nuestro universo parecen estar logrando una hazaña imposible. Están girando a tal velocidad que la gravedad generada por su materia observable no podría mantenerlos juntos; deberían haberse desgarrado hace mucho tiempo. Lo mismo ocurre con las galaxias en cúmulos, lo que lleva a los científicos a creer que algo que no podemos ver está funcionando. Piensan que algo que aún tenemos que detectar directamente es darles a estas galaxias masa adicional, generando la gravedad adicional que necesitan para permanecer intactas. Esta materia extraña y desconocida se llamó “materia oscura” ya que no es visible. Exactamente, ¿cuál es la naturaleza de la “materia oscura”, este misterioso material que ejerce una atracción gravitacional, pero que no emite ni absorbe luz? Los astrónomos no lo saben.

¡La materia bariónica comprende solo el 4.6% de la masa total del universo observable! 24% es frío, materia oscura. Es probable que la materia oscura esté compuesta por una o más especies de partículas subatómicas que interactúan muy débilmente con la materia ordinaria. Otra idea es que podría contener “partículas supersimétricas”, partículas hipotéticas que son asociadas a las que ya se conocen en el Modelo Estándar. Los experimentos en el Gran Colisionador de Hadrones (LHC) pueden proporcionar pistas más directas sobre la materia oscura en los próximos años.


A diferencia de la materia normal, la materia oscura no interactúa con la fuerza electromagnética. Esto significa que no absorbe, refleja o emite luz, por lo que es extremadamente difícil de detectar. De hecho, los investigadores han podido inferir la existencia de materia oscura solo a partir del efecto gravitacional que parece tener sobre la materia visible. .

Muchas teorías dicen que las partículas de materia oscura serían lo suficientemente claras como para ser producidas en el LHC. Si fueran creados en el LHC, escaparían a través de los detectores sin ser notados. Sin embargo, se llevarían la energía y el impulso, por lo que los físicos podrían inferir su existencia a partir de la cantidad de energía y el impulso “que faltan” después de una colisión. Los candidatos a materia oscura surgen con frecuencia en teorías que sugieren física más allá del Modelo Estándar, como la supersimetría y las dimensiones adicionales. Una teoría sugiere la existencia de un “Valle Oculto”, un mundo paralelo hecho de materia oscura que tiene muy poco en común con la materia que conocemos.

Si una de estas teorías resultara cierta, podría ayudar a los científicos a comprender mejor la composición de nuestro universo y, en particular, cómo se mantienen unidas las galaxias.

Materia oscura | CERN

La materia oscura es el nombre dado a la masa adicional cuya presencia se infiere solo de la atracción gravitacional pero que no interactúa con la luz . Representa aproximadamente el 25% de la densidad de energía del universo. Se supone que la materia oscura es una partícula masiva (escala ~ 100 GeV) que básicamente no interactúa, excepto a través de la gravedad. Hay varios esfuerzos en curso para detectar directa o indirectamente la presunta partícula de materia oscura.

Razones por las que creemos en la materia oscura:

  1. Curvas de rotación galáctica : la rotación angular de las galaxias espirales en función del radio le permite a uno medir la masa interior a ese radio. Este método da una masa aproximadamente 10 veces la masa que podemos ver de la materia luminosa.
  2. Masas de cúmulos de galaxias a partir de las velocidades de dispersión de galaxias: si se supone que los cúmulos de galaxias están “virializados”, que es una forma de equilibrio cinético, entonces se puede determinar la masa a partir de las velocidades de las galaxias miembros. Esto da una masa aproximadamente 20 veces la masa del material luminoso en las galaxias.
  3. Las masas de cúmulos de galaxias a partir de gas emisor de rayos X caliente : los cúmulos de galaxias masivas tienen gas caliente y difuso (¡aproximadamente la misma masa que en las galaxias!) Que brilla en la banda de rayos X. Este gas también está cerca de ser virializado, y las mediciones del espectro de rayos X pueden dar una masa de acuerdo con lo anterior.
  4. Lentes gravitacionales : según la relatividad general, la luz se desvía del camino que tomaría en la gravedad newtoniana debido a la presencia de masa o energía. La cantidad de desviación o las estadísticas de las distorsiones de muchas imágenes alrededor de un objeto se pueden usar para medir la masa gravitante total de la lente, independientemente del tipo de masa gravitante. Esto nos permite trazar dónde está la masa y compararla con dónde está la luz. Un ejemplo famoso de esto es el cúmulo de balas, que puede mapearse usando luz visible para determinar dónde están las estrellas; Rayos X, para determinar dónde está el gas intergaláctico caliente; y lentes, para determinar dónde está la mayor parte de la masa. El gas de rayos X es tan masivo o más que las estrellas, y es despojado de las galaxias debido a un reciente evento de fusión. Los datos de las lentes cuentan la historia de que la mayor parte de la masa (que es más que la vista desde la masa de gas de rayos X más las estrellas) no ha colisionado y acaba de pasar justo después de este evento de fusión.
  5. Antecedentes de microondas cósmicos : el análisis del CMB determina la escala de agrupamiento de la materia en el momento de la última dispersión. Hay varios “picos acústicos” en el espectro multipolar del CMB, que corresponden a compresiones y rarefacciones de plasma a escalas correspondientes a semiciclos de la edad del universo en ese momento. Dado que la materia normal tiene soporte de presión, pero la materia oscura en principio no lo haría, esto afectaría las compresiones de diferentes maneras. Los ajustes a los datos requieren materia oscura para ser consistentes con el patrón de compresiones y rarefacciones.
  6. La oscilación acústica de Baryon es similar al efecto anterior, pero en las escalas de longitud del universo actual, no en el momento de la última dispersión.

La supersimetría sugiere que debería haber una “partícula supersimétrica más ligera” que sería un candidato ideal para partículas de materia oscura. Otras propuestas son el axión o cualquier partícula más ligera de algún modelo de teoría de cuerdas . Los modelos excluidos son MACHO (MAssive Compact Halo Objects) y materia oscura de neutrinos (los neutrinos estarían demasiado calientes para agruparse en las escalas apropiadas). Un nombre general para modelos no específicos es WIMP (partículas masivas que interactúan débilmente).

Las búsquedas de detección directa operan bajo el supuesto de que la materia oscura debe tener una sección transversal de interacción muy pequeña que le permita dispersarse del material en un detector que trabajan muy duro para aislar de todas las interacciones externas. Otro enfoque es esperar que a energías muy altas, un experimento de colisionador genere partículas de materia oscura que se mostrarían como falta de impulso / energía en el seguimiento de colisión.

Las detecciones indirectas funcionan bajo el supuesto de que la materia oscura puede tener una pequeña sección transversal de aniquilación y buscan la firma de la aniquilación en regiones de alta densidad de materia oscura.

Las teorías alternativas de la gravedad intentan explicar los efectos anteriores modificando la relatividad general en lugar de introducir una nueva partícula masiva. MOND (Dinámica Newtoniana Modificada) generalmente está desacreditado por ser un modelo fenomenológico en lugar de un modelo fundamental y requiere un ajuste para que coincida con diferentes escalas, además de no poder reproducir datos de lentes, CMB o BAO. TeVeS es otra alternativa que es una teoría fundamental (no sé lo suficiente para comentar sobre su validez).

La materia oscura es parte del modelo estándar de cosmología . Es cierto que hay varios candidatos para la partícula de materia oscura y no estamos seguros de cuál es la correcta. Sin embargo, el campo está bastante seguro de que Dark Matter es un signo de nueva física más allá del modelo estándar de física de partículas, y no de una modificación a la Relatividad General. Esto se debe al alto nivel de consistencia entre los datos mencionados anteriormente, suponiendo que la materia oscura WIMP. Como creemos que debe haber un modelo más allá del modelo estándar, es natural aceptar que hay partículas más masivas que aún no entendemos.

Respuesta corta..

La materia oscura, es la materia que es invisible para nosotros. La razón más simple podría ser que dicha materia no interactúa con la radiación electromagnética. No puede absorber o emitir suficiente radiación para ser detectable con la tecnología de imagen actual


Respuesta larga..

Jan Oort era un astrónomo holandés que estudiaba movimientos estelares. Según una teoría, la masa del plano galáctico debe ser mayor de lo que observó. Más tarde, otro astrónomo zwicky, que estaba haciendo una investigación similar, observó lo mismo. Sabía que había algo que le faltaba. Al hacer una investigación sobre el cúmulo de galaxias Coma, obtuvo evidencia de una masa invisible que llamó “ materia oscura ”.

Más tarde, los científicos concluyeron que la materia oscura es el tipo de materia no identificada que constituye el 27% del universo observable.

Este 27% no tiene en cuenta la energía oscura, los neutrinos o la materia bariónica. La energía oscura es completamente diferente. Es algo que es responsable de la expansión exponencial del espacio. Respuesta de Raveena Dandona a ¿Es el espacio-tiempo realmente plano (una hoja) o está representado con mayor precisión en esta imagen?

Un neutrino es un fermión (partícula elemental con espín entero medio) que interactúa solo a través de la débil fuerza subatómica. Es neutral. No se ve afectado por el campo electromagnético porque su masa en reposo es cero.

La materia bariónica solo incluye la materia compuesta de bariones que incluye protones, neutrones y todos los objetos compuestos por ellos (es decir, núcleos atómicos), pero excluye cosas como electrones y neutrinos que en realidad son leptones.

No emite ni interactúa con radiación electromagnética, como la luz, por lo que es invisible a todo el espectro electromagnético. Es tan denso y pequeño que no puede absorber o emitir suficiente radiación para ser detectable con la tecnología de imagen actual.

La hipótesis de la materia oscura desempeña un papel central en el modelado actual de la formación de la estructura cósmica y la formación y evolución de galaxias y en las explicaciones de las anisotropías observadas en el fondo cósmico de microondas (CMB). Todas estas líneas de evidencia sugieren que las galaxias, los cúmulos de galaxias y el universo en su conjunto contienen mucha más materia que la que se puede observar a través de señales electromagnéticas.

EVIDENCIA

  • Gran parte de la evidencia proviene de los movimientos de las galaxias. Muchas de estas parecen ser bastante uniformes, por lo que, según el teorema virial, la energía cinética total debería ser la mitad de la energía de unión gravitacional total de las galaxias. Observacionalmente, la energía cinética total es mucho mayor.
  • La velocidad de rotación / orbital de las galaxias no disminuye con la distancia, a diferencia de otros sistemas orbitales como estrellas / planetas y planetas / lunas que también tienen la mayor parte de su masa en el centro. Las observaciones en masa de las galaxias basadas en la luz que emiten son demasiado bajas para explicar las observaciones de velocidad. La hipótesis de la materia oscura explica la masa faltante, explicando la anomalía.

  • Las mediciones difusas de gas interestelar de los bordes galácticos indican la falta de materia ordinaria más allá del límite visible, pero esas galaxias están unidas gravitacionalmente y orbitan entre sí con velocidades que corresponden a velocidades orbitales predichas de relatividad general de hasta diez veces sus radios visibles. .
  • Los cúmulos de galaxias son particularmente importantes para los estudios de materia oscura ya que sus masas se pueden estimar de tres maneras independientes
  • De la dispersión en las velocidades radiales de las galaxias dentro de los cúmulos De los rayos X emitidos por el gas caliente en los cúmulos.
  • A partir del espectro y flujo de energía de rayos X, se puede estimar la temperatura y la densidad del gas, por lo tanto, dar la presión asumiendo que el equilibrio de presión y gravedad determina el perfil de masa del grupo.
  • Muchas observaciones de rayos X candra utilizan esta técnica para determinar independientemente las masas de racimo.
  • La lente gravitacional también juega un papel importante.
    • ¡Cómo la lente gravitacional nos muestra la materia oscura!
    • Redshirt distorsión. Prueba de agrupamiento de materia oscura con distorsiones de espacio Redshift

    La hipótesis más ampliamente aceptada sobre la forma de la materia oscura es que está compuesta de partículas masivas que interactúan débilmente (WIMP) que interactúan solo a través de la gravedad y la fuerza débil.

    Detección directa de materia oscura WIMP


    Otras teorías ..

    • Masa en dimensiones extra
    • Esa materia oscura podría agregarse de la misma manera que la materia ordinaria, formando galaxias de otras dimensiones.
    • Defectos topológicos
    • La materia oscura podría consistir en defectos primordiales en la topología de los campos cuánticos, que contendrían energía y, por lo tanto, gravitarían. Esta hipótesis puede investigarse mediante el uso de una red orbital de relojes atómicos que registraría el paso de defectos topológicos por cambios en la sincronización del reloj.
  • Fractalidad espacio-temporal
    • Aplicando la relatividad al espacio-tiempo fractal no diferenciable, Nottale sugiere que la energía potencial puede surgir debido a la fractalidad del espacio-tiempo, lo que explicaría la energía de masa faltante observada a escala cosmológica.

    Soy un principiante y no sé mucho sobre materia oscura y energía oscura. Si alguien desea agregar algo a esta respuesta, las ediciones son bienvenidas.

    La materia oscura es un tipo de materia hipotética que no se puede ver con telescopios, pero representa la mayor parte de la materia en el universo. La existencia y las propiedades de la materia oscura se infieren de sus efectos gravitacionales sobre la materia visible, la radiación y la estructura a gran escala del universo. La materia oscura no se ha detectado directamente, por lo que es uno de los mayores misterios de la astrofísica moderna.

    La materia oscura no emite ni absorbe luz ni ninguna otra radiación electromagnética a ningún nivel significativo. Según el equipo de la misión de Planck, y según el modelo estándar de cosmología, la energía total de masa del universo conocido contiene 4.9% de materia ordinaria (bariónica), 26.8% de materia oscura y 68.3% de energía oscura. Por lo tanto, se estima que la materia oscura constituye el 84.5% de la materia total en el universo, mientras que la energía oscura más la materia oscura constituyen el 95.1% del contenido total de masa-energía del universo.

    Los astrofísicos plantearon la hipótesis de la existencia de materia oscura para explicar las discrepancias entre la masa de grandes objetos astronómicos determinados a partir de sus efectos gravitacionales, y su masa calculada a partir de la materia observable (estrellas, gas y polvo) que se puede ver que contienen. Sus efectos gravitacionales sugieren que sus masas son mucho mayores de lo que sugiere la encuesta de materia observable.

    Pero, ¿qué lo hace tan especial y por qué nos interesa saberlo?

    Las galaxias en nuestro universo parecen estar logrando una hazaña imposible. Están girando a tal velocidad que la gravedad generada por su materia observable no podría mantenerlos juntos; deberían haberse desgarrado hace mucho tiempo. Lo mismo ocurre con las galaxias en cúmulos, lo que lleva a los científicos a creer que algo que no podemos ver está funcionando. Piensan que algo que aún tenemos que detectar directamente es darles a estas galaxias masa adicional, generando la gravedad adicional que necesitan para permanecer intactas. Esta materia extraña y desconocida se llamó “materia oscura” ya que no es visible.

    ¿Por qué es tan difícil inferir su existencia?

    A diferencia de la materia normal, la materia oscura no interactúa con la fuerza electromagnética. Esto significa que no absorbe, refleja o emite luz, por lo que es extremadamente difícil de detectar. De hecho, los investigadores han podido inferir la existencia de materia oscura solo a partir del efecto gravitacional que parece tener sobre la materia visible.

    La analogía dibujada entre la materia oscura y la energía oscura:

    A principios de la década de 1990, una cosa era bastante segura sobre la expansión del Universo. Es posible que tenga suficiente densidad de energía para detener su expansión y reincorporación, podría tener tan poca densidad de energía que nunca dejaría de expandirse, pero la gravedad seguramente ralentizaría la expansión con el paso del tiempo. Por supuesto, la desaceleración no se había observado, pero, en teoría, el Universo tuvo que disminuir. El Universo está lleno de materia y la fuerza de gravedad atractiva atrae a toda la materia. Luego vino 1998 y las observaciones del Telescopio Espacial Hubble (HST) de supernovas muy distantes que mostraron que, hace mucho tiempo, el Universo se estaba expandiendo más lentamente de lo que es hoy. Entonces, la expansión del Universo no se ha desacelerado debido a la gravedad, como todos pensaban, se ha acelerado. Nadie esperaba esto, nadie sabía cómo explicarlo. Pero algo lo estaba causando.

    Finalmente, los teóricos llegaron a tres tipos de explicaciones. Tal vez fue el resultado de una versión descartada por mucho tiempo de la teoría de la gravedad de Einstein, una que contenía lo que se llamó una “constante cosmológica”. Quizás había algún tipo extraño de fluido energético que llenaba el espacio. Quizás haya algo mal con la teoría de la gravedad de Einstein y una nueva teoría podría incluir algún tipo de campo que cree esta aceleración cósmica. Los teóricos aún no saben cuál es la explicación correcta, pero le han dado un nombre a la solución. Se llama energía oscura.

    La materia oscura es invisible. Basado en el efecto de la lente gravitacional, se ha inferido un anillo de materia oscura en esta imagen de un cúmulo de galaxias (CL0024 + 17) y se ha representado en azul

    Y si encuentra aburrido el contenido anterior, sepa que es solo otro término acuñado por los científicos para explicar la presencia de algo que existe.
    En cuanto a su nombre, quiero decir que la materia oscura se llama materia oscura por la misma razón que una manzana se llama manzana.
    ¿Qué hay en un nombre?

    No existe un problema actual de mayor importancia para la cosmología que el de la materia oscura. La materia oscura está compuesta de partículas que no absorben, reflejan o emiten luz, por lo que no pueden detectarse observando radiación electromagnética. La materia oscura es material que no se puede ver directamente. Sabemos que la materia oscura existe debido al efecto que tiene sobre los objetos que podemos observar directamente.

    Los científicos estudian la materia oscura observando los efectos que tiene sobre los objetos visibles. Los científicos creen que la materia oscura puede explicar los movimientos inexplicables de las estrellas dentro de las galaxias. Las computadoras juegan un papel importante en la búsqueda de datos de materia oscura. Permiten a los científicos crear modelos que predicen el comportamiento de las galaxias. Los satélites también se están utilizando para recopilar datos de materia oscura. En 1997, una imagen del telescopio espacial Hubble reveló la luz de un cúmulo de galaxias distante que estaba siendo doblado por otro cúmulo en primer plano de la imagen. Según la forma en que se dobló la luz, los científicos estimaron que la masa del grupo en primer plano era 250 veces mayor que la materia visible en el grupo. Los científicos creen que la materia oscura en el grupo representa la masa inexplicable.

    La lente gravitacional creada por Galaxy Cluster revela la presencia de materia oscura

    Los científicos han producido muchas teorías sobre qué puede ser exactamente la materia oscura. Algunos creen que pueden ser objetos normales, como gases fríos, galaxias oscuras u objetos de halo compactos masivos (llamados MACHO, incluirían agujeros negros y enanas marrones). Otros científicos creen que la materia oscura puede estar compuesta de partículas extrañas que se crearon en el universo primitivo. Dichas partículas pueden incluir axiones, partículas masivas que interactúan débilmente (llamadas WIMP) o neutrinos.

    Comprender la materia oscura es importante para comprender el tamaño, la forma y el futuro del universo. La cantidad de materia oscura en el universo determinará si el universo está abierto (continúa expandiéndose), cerrado (se expande hasta un punto y luego colapsa) o plano (se expande y luego se detiene cuando alcanza el equilibrio). Comprender la materia oscura también ayudará a explicar definitivamente la formación y evolución de galaxias y cúmulos. A medida que una galaxia gira, debe romperse. Esto no sucede, por lo que algo mantiene la galaxia unida. El algo es la gravedad; Sin embargo, la cantidad de gravedad requerida para hacer esto es enorme y no puede ser generada por la materia visible en la galaxia.

    Aproximadamente el 80 por ciento de la masa del universo está compuesto de material que los científicos no pueden observar directamente. Conocido como materia oscura, este extraño ingrediente no emite luz ni energía.

    Los estudios de otras galaxias en la década de 1950 indicaron por primera vez que el universo contenía más materia que la vista a simple vista. El apoyo a la materia oscura ha crecido, y aunque no se ha detectado evidencia directa sólida de materia oscura, ha habido grandes posibilidades en los últimos años.

    “Los movimientos de las estrellas te dicen cuánta materia hay”, dijo en un comunicado Pieter van Dokkum, investigador de la Universidad de Yale. “No les importa de qué forma se trata, simplemente te dicen que está ahí”. Van Dokkum dirigió un equipo que identificó la galaxia Dragonfly 44, que está compuesta casi por completo de materia oscura.

    El material familiar del universo, conocido como materia bariónica, está compuesto de protones, neutrones y electrones. La materia oscura puede estar hecha de materia bariónica o no bariónica. Para mantener unidos los elementos del universo, la materia oscura debe constituir aproximadamente el 80 por ciento de su materia. [Galería de imágenes: Dark Matter Across the Universe]

    La materia faltante podría ser simplemente más difícil de detectar, compuesta de materia regular y bariónica. Los posibles candidatos incluyen enanas marrones tenues, enanas blancas y estrellas de neutrinos. Los agujeros negros supermasivos también podrían ser parte de la diferencia. Pero estos objetos difíciles de detectar tendrían que desempeñar un papel más dominante de lo que los científicos han observado para formar la masa faltante, mientras que otros elementos sugieren que la materia oscura es más exótica.

    La mayoría de los científicos piensan que la materia oscura está compuesta de materia no bariónica. El candidato principal, WIMPS (partículas masivas que interactúan débilmente), tiene de diez a cien veces la masa de un protón, pero sus interacciones débiles con la materia “normal” hacen que sean difíciles de detectar. Los neutralinos, partículas hipotéticas masivas más pesadas y más lentas que los neutrinos, son los principales candidatos, aunque aún no se han detectado.

    Los neutrinos estériles son otro candidato. Los neutrinos son partículas que no forman materia regular. Un río de neutrinos fluye del sol, pero debido a que rara vez interactúan con la materia normal, atraviesan la Tierra y sus habitantes. Hay tres tipos conocidos de neutrinos; un cuarto, el neutrino estéril, se propone como candidato a la materia oscura. El neutrino estéril solo interactuaría con la materia regular a través de la gravedad.

    “Una de las preguntas pendientes es si hay un patrón en las fracciones que se aplican a cada especie de neutrino”, dijo a Space.com Tyce DeYoung, profesor asociado de física y astronomía en la Universidad Estatal de Michigan y colaborador en el experimento IceCube.

    El axión neutral más pequeño y los fotinos sin carga también son marcadores de posición potenciales para la materia oscura.

    Según una declaración del Laboratorio Nacional Gran Sasso en Italia (LNGS), “Varias mediciones astronómicas han corroborado la existencia de materia oscura, lo que ha llevado a un esfuerzo mundial para observar directamente interacciones de partículas de materia oscura con materia ordinaria en detectores extremadamente sensibles, lo que confirmaría su existencia y arrojaría luz sobre sus propiedades. Sin embargo, estas interacciones son tan débiles que han escapado a la detección directa hasta este punto, obligando a los científicos a construir detectores que son cada vez más sensibles “.

    Existe una tercera posibilidad: que las leyes de la gravedad que hasta ahora han descrito con éxito el movimiento de los objetos dentro del sistema solar requieren revisión.

    Materia oscura .

    Estas son la materia invisible llamada materia oscura.

    ¿Por qué invisible?

    Debido a que no interactúan con la luz y solo vemos las cosas a través de la luz. Ahora, pensemos en el movimiento del aire, no podemos verlo, pero podemos sentir su presencia y sus efectos. Lo mismo ocurre con la materia oscura, no podemos verla pero podemos sentir su presencia y su efecto.

    Supongamos que hay cúmulos de galaxias (las galaxias son como chicas malas y a menudo se encuentran en cúmulos). Cuando las galaxias se orbitan entre sí, a menudo se mueven muy rápido y cuanto más rápido se mueven, se requiere más fuerza para mantenerlas juntas. La fuerza que los mantiene unidos es la gravedad. Y sabemos que más gravedad solo proviene de más masa.

    Pero cuando los científicos vieron, descubrieron que no hay suficiente masa presente para mantener las galaxias juntas, pero aún así las galaxias se están orbitando entre sí. Entonces, el científico asumió que debe haber alguna materia invisible que los mantenga unidos y nombró a esa materia oscura.

    ¿Sabía que nuestro universo se compone de 5% de materia bariónica (por materia bariónica, me refiero a la materia visible como el planeta, las estrellas y todas las demás materias visibles combinadas juntas solo representan el 5%) y el 26% de materia oscura y el 69% de oscuridad energía.

    Gracias por leer y feliz lectura.

    Has recibido muchas respuestas que dan una variedad de perspectivas sobre DM. Aquí está el mío:

    La materia oscura nos proporciona un factor hipotético que expresa el contenido del universo que los modelos matemáticos dicen que debe estar presente para explicar por qué las galaxias no se separan. Los estudios muestran que no hay suficiente masa en la mayoría de las galaxias grandes para mantenerlas juntas y para dar cuenta de sus velocidades de rotación, lentes gravitacionales y otras observaciones de grandes estructuras. La cantidad de masa necesaria para dar cuenta de las observaciones no solo está un poco apagada. En 1933, Fritz Zwicky calculó que tomaría 400 veces más masa de lo que se observa en galaxias y cúmulos para dar cuenta del comportamiento observado. Este no es un número pequeño. La materia oscura representa el 22% de toda la materia en el universo. Como Zwicky confiaba en que sus matemáticas y observaciones eran perfectas, concluyó que, de hecho, hay toda la masa necesaria en cada galaxia, pero simplemente no podemos verla. Así nació el concepto de materia oscura.

    La DM puede detectarse por sus efectos gravitacionales que tienen varias implicaciones. Eso significa que donde el DM tiene una densidad suficientemente alta, podemos ver fuertes efectos de lentes gravitacionales. Por el contrario, significa que en áreas donde no es tan denso, tenemos cada vez menos capacidad para detectarlo. Por ejemplo, es solo una suposición de que hay DM a nuestro alrededor aquí en la tierra. Es solo una conjetura basada en la idea de que podemos detectarlo en otro lugar, pero nuestra capacidad para detectarlo depende tanto de sus efectos de lente que hay vastas áreas del espacio que no nos muestran ningún DM presente. Lo que podemos ver y mapear aparece como filamentos de una gran red donde los nodos de la red son galaxias y densos campos de DM.

    Otra implicación es que la DM tiene una masa que presenta uno de los muchos problemas: a pesar de determinar que la DM es lo suficientemente densa como para tener suficiente gravedad como para alterar las órbitas de las estrellas e incluso las galaxias, es transparente. No conocemos ninguna otra materia que tenga masa y pueda generar tanta gravedad y aún así ser transparente y no emitir nada más.

    Aunque podemos ver 2.7 x 10 ^ 23 millas en el espacio, todavía no hemos observado ni detectado una sola pieza de materia oscura. Para dar cuenta de este hecho aparentemente sorprendente, los defensores dicen: “bueno, duh, es un asunto OSCURO”, ¡no puedes VERLO! “. Sin embargo, parece que no solo es oscuro sino también completamente transparente porque las áreas de materia oscura densa no impiden que las estrellas sean visibles detrás de la materia oscura. Entonces, el 22% de toda la masa en el universo no se puede ver, es, de hecho, transparente, nunca se ha observado, y no parece haber tenido interacciones directas con ninguna masa conocida que no sean los efectos de la gravedad. Esa es, al menos, la comprensión generalmente aceptada.

    Las declaraciones sobre DM que no sean masa, gravedad y pueden detectarse mediante lentes son especulaciones. Por ejemplo, ni siquiera sabemos si es una partícula o no. Los atributos que algunos le otorgan son imaginación total, como la que dice que DM mató a todos los dinosaurios. Cuando lea estas otras respuestas, asegúrese de tener en cuenta lo que realmente sabemos sobre DM y compárelo con lo que se dice al respecto.

    …Solo es mi opinión…

    No sabemos qué es la materia oscura o la energía oscura. Ellos son: nombres de lugar para la causa de ciertos efectos que notamos.

    Efecto: la velocidad de rotación de las estrellas hacia el borde de otras galaxias no está de acuerdo con la velocidad de rotación que obtendríamos de las ecuaciones de Einstein.
    Aquí A es la velocidad de rotación que predice el GR de Einstein, B es la velocidad real medida. Es la gran diferencia.
    Probablemente causa:

    • GR simplemente falla de alguna manera (poco probable, ya que parece funcionar bien en otros lugares, pero esto siempre es algo que debe tener en cuenta.
    • No incluimos los valores correctos de masa, energía, etc. en nuestras ecuaciones, por lo que, por supuesto, obtenemos la respuesta incorrecta (esta parece ser la opción más probable: antes de deshacerse de su buena teoría de trabajo, debe descubrir si realmente lo está utilizando de la manera en que lo justifica la situación)

    Si observa la ecuación newtoniana para la fuerza que siente una partícula giratoria, la fuerza centrípeta. Notamos que tiene la forma de:
    [matemáticas] F_c = \ frac {mv ^ 2} {r} [/ matemáticas]. Si bien la versión de Einstein es más complicada, podemos ver las variables en esta ecuación.
    Tenemos [matemáticas] m [/ matemáticas] la masa en la galaxia.
    [matemáticas] v [/ matemáticas] la velocidad de las estrellas giratorias.
    [matemáticas] r [/ matemáticas] la distancia entre las estrellas y el centro de esa galaxia.
    Tanto [math] v [/ math] como [math] r [/ math] se pueden medir con bastante facilidad, solo necesita mirar la galaxia por un tiempo.
    Entonces, el culpable más probable es [math] m [/ math]: usamos el valor incorrecto (y la distribución) de la masa.

    Podemos medir la masa de una galaxia utilizando otro efecto de GR: la luz se dobla en un campo gravitacional, cuanto mayor es la masa, más fuerte es la flexión (lente gravitacional). Podemos usar esta técnica para calcular no solo la masa total de una galaxia, sino también la distribución de esta masa.

    Como resultado, esta masa se distribuye de manera bastante uniforme en una galaxia (es por eso que dicen que está en todas partes), pero hay varios picos en la distribución de masa (si no recuerdo mal). Sin embargo, estos picos no se corresponden con lo que vemos como materia (agujeros negros, estrellas, nebulosas, etc.). Además, dado que el espacio es principalmente vacío: no hay suficiente materia para afectar realmente esta lente gravitacional, ahora podemos llegar a la conclusión de que hay:

    • Más materia en la galaxia de lo que pensábamos originalmente
    • No podemos ver este asunto: es invisible, oscuro.
    • Toda la materia que sabemos reacciona con la luz, por lo que este nuevo tipo de materia tiene que ser fundamentalmente diferente de otra materia.

    Entonces tenemos materia, y está oscuro. Los físicos somos bastante fáciles con los nombres, lo llamamos materia oscura.

    Ahora, para la energía oscura:

    El universo se está expandiendo, esto es de esperar si asumes un Big Bang: todo sigue volando lejos de todo lo demás, pero si comienzas a obtener los números, encuentras algunas cosas extrañas:
    Efecto: la expansión no se está desacelerando (como era de esperar, ya que toda la materia atrae a la otra materia). No, esta expansión está aumentando.
    Posibles causas:

    • Nuestros datos están equivocados. Si bien esto es posible, no hay efectos conocidos que puedan deformar los datos de tal manera que obtengamos este resultado extraño (pero la gente lo está investigando)
    • Hay otra fuerza empujando todo lejos.

    Por pura coincidencia estúpida, hay un término en las ecuaciones de Einstein que hace exactamente lo que vemos: obliga al espacio a expandirse. Por lo tanto, parece posible (y tal vez probable) que este término realmente necesite estar presente en los cálculos.
    Así que tenemos algo, algo de fuerza, que está alejando a los demás. Tampoco tenemos idea de qué está causando esta fuerza, y mirando hacia atrás en la materia oscura, ¿por qué no llamarla fuerza oscura? Bueno, probablemente puedas ver que ese nombre no sería bueno para los fanáticos de Star Wars. Entonces lo llamaron Energía Oscura, ya que la energía y las fuerzas están estrechamente relacionadas y el término en las ecuaciones de Einstein en realidad representa energía. (No tengo idea si realmente fue así, pero no me sorprendería que no fueran a Dark Force por Star Wars).

    Dado que tenemos poca o ninguna comprensión de lo que realmente son Dark Energy y Dark Matter, son simplemente nombres de marcador de posición para la causa de los efectos que describí. Podrían ser bastante fundamentales, o podría ser un descuido gigante de nuestra parte. En el primer caso, tenemos nuevos descubrimientos para hacer. En el último caso, los nombres eventualmente desaparecerán en la historia, junto con todos los demás nombres que no tenían sentido.

    Como último punto, permítanme ser franco: es bastante estúpido creer solo en las cosas que han visto o notado.

    • Nunca has estado lo suficientemente alto sobre la tierra para notar que el mundo es una esfera (obloide).
    • Probablemente no has estado en Australia ni en Northpole.
    • Nunca ha visto electrones, protones, neutrones o incluso moléculas (que no sean polímeros).
    • Probablemente nunca has visto una cadena de ADN
    • Las bacterias son bastante difíciles de detectar a simple vista.
    • Hay un espectro gigante de ondas electromagnéticas, pero solo se ve una banda muy pequeña.

    Si bien es posible que haya visto algunos de estos, hay toneladas de cosas que acepta sobre nuestro mundo que nunca ha visto. Si bien nunca te pediría que confíes ciegamente en las personas cuando hacen un reclamo, debes tener al menos algún conocimiento sobre el tema para refutarlo.

    Nadie sabe.

    Tiene masa gravitacional. Por lo tanto, afecta otras cosas gravitacionalmente y se ve afectado por la gravedad. La gravedad es una fuerza muy débil, necesitas cosas realmente grandes como planetas, estrellas y galaxias para generar una gravedad significativa.

    No interactúa con los fotones. Las fotos lo atraviesan o lo pasan como si no estuviera allí. Esto casi con certeza significa que no tiene propiedades eléctricas o magnéticas. Esta invisibilidad es la razón por la que tardó tanto en darse cuenta.

    Se cree que comprende aproximadamente 5/6 de la materia en el universo. ¡Hay mucho de eso! La evidencia de la materia oscura son sus efectos gravitacionales. Por ejemplo, las galaxias giratorias se volarían si estuvieran hechas de las cosas que podemos ver: estrellas, polvo, etc. Sus estrellas se mueven en curvas demasiado apretadas para las fuentes de gravedad conocidas. Esto significa que las galaxias no son solo estrellas y polvo, sino también esta materia oscura invisible. Estos efectos han sido observados y medidos. Los físicos confían en que hay algo allí, e incluso pueden hacer una buena estimación de cuánto. Pero eso no nos dice qué es.

    Aislar la materia oscura va a ser difícil. La gravedad es tan débil que no hay esperanza de ver sus efectos a nivel de partículas, y la gravedad parece ser la única forma de interactuar con estas cosas. No podemos acelerar la materia oscura en el LHC porque no responde a las fuerzas electromagnéticas. No hay evidencia de que la materia ordinaria se convierta en materia oscura en colisiones de partículas, pero esto no se puede descartar por completo.

    El otro problema en el lado teórico es que no hay partículas o partículas candidatas obvias que puedan ser materia oscura. Se requerirá una nueva partícula (s) y una revisión del Modelo Estándar. En esta etapa, nadie sabe con certeza cuál podría ser la partícula, aunque hay algunas partículas candidatas, vea los cinco principales candidatos para ‘materia oscura’ (edite, gracias Greg Wadleigh). Tener una estimación de energía para la partícula de materia oscura, como algunos de estos hacen – da una pista de dónde buscar signos de la partícula.

    Lo que sabemos con certeza sobre la materia oscura es:

    • Las partículas de materia oscura tienen una masa en reposo.
    • No tienen fuertes interacciones nucleares o electromagnéticas.
    • Se mueven de manera no relativista, es decir, su energía cinética es mucho menor que su energía de masa en reposo.
    • Constituyen aproximadamente el 23% del contenido de masa / energía del universo, aproximadamente 5 veces más que la masa de la materia ordinaria que forma las estrellas y los planetas.
    • Cuando el universo tenía solo 380,000 años, la materia oscura, la materia ordinaria y la radiación se extendieron muy suavemente por todo el universo con fluctuaciones de solo 1 parte en 10,000 más densas o menos densas que la densidad promedio.
    • En el momento actual, las partículas de materia oscura se han agrupado debido a su gravedad mutua para formar halos masivos alrededor de todas las galaxias y cúmulos de galaxias. En realidad, los cúmulos de materia oscura se formaron primero y luego atrajeron la materia ordinaria para crear galaxias y cúmulos de galaxias.

    Lo que no sabemos sobre las partículas de materia oscura son:

    • su masa en reposo,
    • si participan en las interacciones nucleares débiles y
    • si solo hay uno o más de un tipo de partícula de materia oscura.

    Existen dos modelos posibles que se sugieren teóricamente como una posible partícula de materia oscura a partir de ahora:

    • Podrían ser masivos, como 10 a 1000 veces la masa de un protón, esta es la hipótesis de la materia oscura WIMP (Partícula masiva débilmente interactiva).
    • Podrían ser muy ligeros, como una fracción de un eV, esta es la hipótesis de la partícula de materia oscura de Axion

    Todos los físicos esperamos que la partícula de materia oscura realmente participe en las interacciones nucleares débiles, de lo contrario no tendríamos forma de detectarlas directamente. Si solo tienen interacciones gravitacionales, las partículas individuales nunca serían detectables.

    Cuando los científicos querían descubrir a qué velocidad giraban las galaxias, descubrieron que las galaxias están girando a una velocidad mucho más alta de lo que debería ser. El valor real fue mucho más alto que el valor calculado.

    Cuando cualquier cuerpo gira sobre otro cuerpo, la velocidad de rotación depende de la fuerza gravitacional que actúa sobre él. A mayor fuerza gravitacional, mayor será la velocidad de rotación.

    Por lo tanto, los científicos se confundieron sobre el valor y asumieron que debía haber algo de donde provenía esta enorme fuerza gravitacional.
    Pero dondequiera que se enfocaran en el espacio, no podían encontrar ningún asunto. Entonces solo le dieron un nombre.
    Materia oscura.

    Un tipo especial de materia que no refleja la luz, absorbe la luz y refracta la luz.

    Cuando calcularon la cantidad total de materia oscura presente en todo el universo, se descubrió que el 23% de toda la energía del universo es materia oscura. Donde todas las galaxias están formadas solo por un 4%.

    La materia oscura es un nombre algo ambiguo dado a un conjunto de observaciones astronómicas que las teorías físicas que no incluyen la materia oscura no explican. Estas observaciones tienden a sugerir la presencia de masa que no podemos ver, de ahí la elección de la “materia oscura”.

    Por ejemplo, si mide la velocidad de un grupo de estrellas que se mueven alrededor del centro de una galaxia, y luego grafica esas velocidades en función de la distancia desde el centro, obtiene lo que se llama una curva de rotación de galaxias. Las velocidades de las estrellas son una función de la cantidad de masa contenida en la galaxia. Podemos medir cuánta masa hay en la galaxia en función de su luminosidad y combinando eso con lo que sabemos sobre astrofísica galáctica. Sin embargo, las velocidades de las estrellas que medimos en realidad son mayores que las predichas por la masa de la galaxia, lo que implica una masa extra alrededor del exterior que no vemos. Esta es la “materia oscura”.
    Tienden a parecerse a esto.

    La historia de la “materia oscura” es realmente fascinante. Entre mediados y finales del siglo XX, se comenzaron a utilizar muchas tecnologías y técnicas nuevas en astronomía, y una de las cosas que permitieron fue poder medir la masa de galaxias por separado de la estimación de su masa en función de su población observada de estrellas. Esto condujo a un resultado notable, las galaxias y los cúmulos de galaxias parecían tener más masa de la que podría ser explicada por las estrellas, mucho más, muchas veces más. Se expusieron muchas teorías para explicar esto, desde errores experimentales hasta gases y polvo interestelares, planetoides deshonestos, mini agujeros negros, neutrinos y partículas subatómicas aún más exóticas aún desconocidas.

    La continuación de la historia es aún más notable. La evidencia seguía llegando de una variedad de fuentes diferentes (vea el artículo de wikipedia para obtener una lista), la masa extra era real. Aún más extraño, no podría ser gas, polvo, planetoides, agujeros negros o incluso neutrinos. No podían ser átomos porque cada teoría que se nos ocurrió que tenía esta masa extra compuesta de átomos no coincidía con las observaciones, y no en gran medida, en gran medida. A pesar de todas las probabilidades, la única teoría que se ajustaba a toda la evidencia era que esta “materia oscura” estaba compuesta de partículas que apenas interactúan con la materia atómica. Eso no es del todo sin precedentes, ya que ya sabemos acerca de los neutrinos que tienen muchas de las mismas propiedades, pero también descartamos a los neutrinos mismos como la fuente de la materia oscura, que dejó partículas que aún no habíamos descubierto.

    Y ahí es donde nos encontramos hoy. No sabemos qué es realmente la materia oscura, aparte de que es casi seguro que es algún tipo de partícula que aún no hemos detectado directamente y que no tenemos la física para describir en este momento. Existen teorías de física de partículas que incluyen tales partículas y, de hecho, la mayoría de las extensiones razonables del “modelo estándar” existente de física de partículas terminaría incluyendo partículas de esa naturaleza, pero aún tenemos que verificar tales teorías.

    La buena noticia es que entre la investigación de física de partículas que utiliza colisionadores de partículas de alta energía, las observaciones del telescopio espacial de rayos gamma y los experimentos de detección de materia oscura en la Tierra, tenemos una probabilidad bastante alta de determinar la naturaleza exacta de las partículas de materia oscura en las próximas décadas.
    Además, una faceta interesante de la teoría líder actual de la composición de la materia oscura es que debería ser una partícula que solo se crearía en condiciones extremadamente energéticas que explicarían su abundancia. Dado que tales condiciones habrían existido durante los primeros momentos del Universo justo después del Big Bang, causando que cantidades masivas de masa / energía se convirtieran en partículas efímeras de materia oscura que luego dejaron de interactuar con todo lo demás y luego el Universo se enfrió lo suficiente como para que tales partículas ya no se podían producir, y la masa / energía restante se convirtió en átomos y fotones, etc.

    ¿Qué es la materia oscura?

    Propongo que la materia oscura es una deficiencia de energía oscura. La energía oscura consiste en neutrinos y partículas simétricas pronosticadas de tipo neutrino. Dominan la dinámica a gran escala de nuestro universo. Están en todas partes y aceleran todo lejos de ellos, incluida la mayor parte de nuestro universo. La gravedad positiva da como resultado energías potenciales gravitacionales negativas. Por el contrario, la gravedad negativa da como resultado energías potenciales gravitacionales positivas. Hay una región que rodea las galaxias unidas gravitacionalmente donde los neutrinos de energía oscura dominan sobre la materia ordinaria y la energía potencial gravitacional cambia de negativa a positiva. Para explicar la materia oscura, los neutrinos de energía oscura deben ser deficientes en galaxias. La energía potencial sería menor donde los neutrinos de energía oscura son deficientes. Todo tiene un componente de aceleración hacia estas deficiencias.

    Se suponía que la materia oscura era una partícula desconocida, en lugar de una deficiencia de energía oscura. Esto condujo a una estimación excesiva de la cantidad de energía oscura requerida para superar la materia ordinaria y la materia oscura, que sería suficiente para causar la aceleración de nuestro universo. Unificar la materia oscura y la energía oscura es frugal. Reduce las cantidades de partículas desconocidas requeridas para dar cuenta de estos efectos oscuros. No se requeriría materia oscura en absoluto. Suponiendo que existiera materia oscura, se pronostica que nuestro universo será 4.9% de materia ordinaria y 26.8% de materia oscura, para un total de 31.7% de materia de gravedad positiva; y 68.3% de gravedad negativa de energía oscura. Si no hay materia oscura, tanto la materia de gravedad positiva total como la energía oscura de gravedad negativa requerida para superarla se reducen en un factor de: 31.7% / 4.9% = 6.5. Esto reduce la energía oscura requerida a solo 1 / 6.5 = 0.155 o 15.5% de lo que se creía anteriormente. Sin materia oscura, nuestro universo tiene 67.7% de energía oscura y 31.8% de materia ordinaria. Ahora debería haber suficientes neutrinos y partículas simétricas de tipo neutrino predichas para dar cuenta de la energía oscura.

    La simetría de masa relativista es la simetría entre la materia positiva y la materia negativa (sí, negativa). Es muy improbable que pueda existir entre masas positivas. La materia ordinaria, que vemos, tiene masas relativistas positivas-reales (mγ), masas en reposo (m), factores de Lorentz (γ), energías [E = c ^ 2mγ], pero aceleraciones gravitacionales predichas newtonianas negativas [a = –mγ x (vector r / r) / (r ^ 2)] porque mγ es positivo-real. Los neutrinos de simetría de masa relativista son la energía oscura hipotética, no se pueden ver, tienen masas relativistas negativas (m’γ ‘= – mγ), masas de descanso imaginarias negativas (m’ = – mγ / γ ‘), factores de Lorentz (γ’ = –Ix> 10 ^ 6), y aceleraciones gravitacionales predichas newtonianas positivas [a ‘= –m’γ’ x (vector r / r) / (r ^ 2)] porque m’γ ‘es negativo-real y las dobles negativas volverse positivo

    Descubrí la “simetría de masa relativista” al investigar la masa relativista negativa (m’γ ‘) como candidato para la energía oscura y descubrí que los neutrinos tenían sus propiedades predichas. La simetría de masa relativista no solo predice los neutrinos y sus propiedades, sino también nuevas partículas similares a los neutrinos. Aquí está la ecuación de predicción precisa para las masas en reposo de los neutrinos y las partículas pronosticadas similares a las de los neutrinos, en términos de las masas en reposo de las partículas del mismo nombre:

    m ‘= – mγ / γ’,

    donde m ‘es la masa de reposo imaginaria negativa de los neutrinos; m es la masa en reposo de la partícula ordinaria homónima correspondiente de los neutrinos; γ es el factor de Lorentz de la partícula ordinaria homónima correspondiente de los neutrinos, que normalmente es esencialmente igual a un γ ≈ 1.0; y γ ‘es el factor de Lorentz negativo-imaginario de los neutrinos, que siempre es γ’ = –ix> 10 ^ 6. Las masas de descanso de neutrinos pronosticadas con precisión resultantes son:

    Masa de reposo del neutrino electrónico = –ix <0.511 eV / c ^ 2,
    Masa de reposo del neutrino muón = –ix <105.7 eV / c ^ 2,
    Masa de reposo de tau neutrino = –ix <1.777 eV / c ^ 2.

    Las masas de descanso previstas de los antineutrinos son las mismas. Estos valores pronosticados son algo menores que sus valores límite superiores publicados, calculados a partir de sus valores de masa al cuadrado (m ‘^ 2) medidos negativos (sí, negativos). La raíz cuadrada de un número negativo es ± un número imaginario (sí, imaginario). Las masas de neutrinos de electrones observadas actualmente son inferiores a 1 eV y sus energías son de al menos 1 MeV, por lo que sus factores de Lorentz (γ ‘) calculados a partir de la relación <1 eV /> 1 MeV => 10 ^ 6 son: γ’ ≡ 1 / √ [1 – (v ‘/ c) ^ 2] = –ix> 10 ^ 6. Las velocidades | v ‘| de los neutrinos observados actualmente se puede calcular con precisión a partir de estos factores de Lorentz. Están en el rango de: c <| v '| <(1 + 0.5 x 10 ^ –12) c. El último diferencial de velocidad promedio medido por CNGS2 (ICARUS) fue: (| v '| - c) = (0.04 ± 0.28stat. ± 0.98syst.) X 10–6 c, que es más rápido (sí, más rápido) que la velocidad de luz en el vacío. El primer postulado de la relatividad especial, el principio de la relatividad, indica que las propiedades múltiples de los neutrinos requieren un viaje a mayor velocidad que la luz en el vacío. (1) Debido a que los neutrinos nunca se detienen en ninguno de nuestros marcos de referencia que viajan a menos de la velocidad de la luz, y tienen masa para que no puedan viajar a la velocidad de la luz, deben viajar a una velocidad mayor que la de la luz en un vacío, donde pueden quedarse quietos en su marco de referencia. (2) Si los neutrinos viajaran a menos de la velocidad de la luz en el vacío, habría marcos de referencia en los que cambiarían la helicidad. Pero todos los neutrinos tienen helicidad zurda, nunca cambian la helicidad. (3) Para que el principio de relatividad y “simetría de masa relativista” funcione entre ambos electrones y sus neutrinos de electrones del mismo nombre, uno de ellos debe viajar a una velocidad superior a la velocidad de la luz en el vacío. Las leyes de la física funcionan en sus marcos de referencia y en nuestros marcos de referencia. En sus marcos de referencia, nosotros somos los que viajamos siempre a mayor vacío que la semilla de la luz.

    La energía oscura está compuesta de estas partículas de simetría de masa relativista. Por otro lado, la materia oscura, he propuesto, es una deficiencia de estos neutrinos de energía oscura. Al no tener partículas de materia oscura, reduce la energía oscura requerida en un factor de 6.5. Nuestro nuevo universo frugal ahora contiene tres tipos diferentes de masa en reposo: negativo-imaginario, cero y positivo; proporcionado de la siguiente manera: 67.7% de energía oscura, 31.8% de fotones de luz y materia ordinaria. Mis predicciones revolucionarias se basan sólidamente en la teoría especial de la relatividad y los datos de observación de Einstein. Dado que muchos neutrinos ya están en el modelo estándar de la mecánica cuántica, la energía oscura y la materia oscura ya no son partículas oscuras desconocidas. La física, la astronomía y la cosmología ahora se basan sólidamente en partículas cuánticas mecánicas conocidas y predichas.

    PD: La serie Science Channel, “Los secretos más profundos del espacio”, Temporada 1, Episodio 44, “La caza de la energía oscura”, sugiere que cualquiera que descubra la respuesta a “¿Qué es la energía oscura?” Recibiría un Premio Nobel de física. Descubrí esa respuesta y propuse una respuesta a, “¿Qué es la materia oscura?” Aquellos de ustedes que encuentran mis respuestas revolucionarias persuasivas o incluso interesantes, por favor “Voten” en Quora. Necesitamos hacer todo lo posible para avanzar en esta revolución en la física. Y eso incluye que recibí ese Premio Nobel.

    More Interesting

    ¿Por qué el CMBR no es uniforme?

    ¿Cuáles son los problemas con la explicación alternativa para la energía oscura del Prof. Edward Kipreos?

    ¿Podría haber vida hecha de materia oscura y energía?

    ¿Por qué los agujeros negros supermasivos siempre ocurren en el centro de cualquier galaxia?

    ¿Cuáles son las teorías más comunes sobre el 'borde' del universo?

    ¿Existe fricción en el espacio exterior con respecto a que hay materia, antimateria, materia oscura, energía oscura y energía negativa también?

    Si todo en el universo está hecho de las mismas cosas, ¿significa que cada cosa en el universo está muerta o cada cosa en el universo está viva?

    ¿Quién o qué creó el universo en el que vivimos?

    ¿Cuál sería la masa aproximada de un agujero negro?

    ¿Por qué vemos la misma imagen (CMB) en todas las direcciones y por qué da la ilusión de que estamos en el centro?

    En un futuro lejano, ¿por qué no podríamos ver la luz de las galaxias que podemos ver ahora, si el horizonte de partículas se expande al mismo ritmo que el universo?

    ¿Qué significa que nuestro universo es finito e ilimitado?

    En el universo primitivo, en la época del Big Bang, ¿se estaba "creando" el espacio o simplemente se estaba "expandiendo"?

    ¿Por qué no puede haber seres vivos del tamaño de galaxias?

    Como no podemos ver la materia oscura, ¿por qué se supone que las fuerzas gravitacionales adicionales son causadas por una forma de 'materia'? ¿No es tan probable que sea algo completamente diferente?